LSAT and Law School Admissions Forum

Get expert LSAT preparation and law school admissions advice from PowerScore Test Preparation.

 Administrator
PowerScore Staff
  • PowerScore Staff
  • Posts: 8916
  • Joined: Feb 02, 2011
|
#25925
Complete Question Explanation
(The complete setup for this game can be found here: lsat/viewtopic.php?t=5457)

The correct answer choice is (A)

The question stem establishes that S lectures second and Z lectures fourth. Since T lectures before S (second rule), we can conclude that T lectures first. Likewise, V lectures before Z (third rule), forcing V into the third position:
oct12_game_2_#9_diagram_1.png
Among the remaining three variables, P lectures before W (first rule), whereas Y does not lecture seventh (fifth rule). Therefore, W must lecture seventh:
oct12_game_2_#9_diagram_2.png
The only uncertainty in the game, then, is the order in which P and Y deliver their lectures. Thus, in a Could Be True question such as this one, you should immediately seek answer choices that contain either P or Y. Only answer choices (A) and (E) qualify, and answer choice (A) is the only one that could be true.

Get the most out of your LSAT Prep Plus subscription.

Analyze and track your performance with our Testing and Analytics Package.